Akademisyenler öncülüğünde matematik/fizik/bilgisayar bilimleri soru cevap platformu
2 beğenilme 0 beğenilmeme
749 kez görüntülendi

Herhangi  $a,b,c$ reel sayıları için \[ |ab(a^2-b^2)+bc(b^2-c^2)+ca(c^2-a^2)|\le M (a^2 + b^2 + c^2)^2 \] eşitsizliğini doğru yapan en küçük $M$ sayısını bulun.

Orta Öğretim Matematik kategorisinde (1.8k puan) tarafından 
tarafından düzenlendi | 749 kez görüntülendi

Kaynak: 2006 IMO, Problem 3.

3 Cevaplar

2 beğenilme 0 beğenilmeme


Warning: imagecreatetruecolor() [function.imagecreatetruecolor]: Invalid image dimensions in /home/salih1/public_html/qa-include/util/image.php on line 145

Warning: imagecolorallocate() expects parameter 1 to be resource, boolean given in /home/salih1/public_html/qa-include/util/image.php on line 146

Warning: imagefill() expects parameter 1 to be resource, boolean given in /home/salih1/public_html/qa-include/util/image.php on line 147
image

(1.8k puan) tarafından 
Son kısmı da ekleyelim. Çözümün tam olması için eşitlik durumu analizi de yapılmalıdır. Eşitlik durumu $(a,b,c)=\left(-1, -1-\dfrac{3}{\sqrt{2}}, -1+\dfrac{3}{\sqrt{2}} \right)$ üçlüleri, bunların katları veya permütasyonları ile elde edilebilir.
2 beğenilme 0 beğenilmeme

yavuzkiremici nin çözümü:

$  ab(a^2-b^2)+bc(b^2-c^2)+ac(c^2-a^2)$ açıp tekrar düzenlersek $(a-c)(a-b)(b-c)(a+b+c)$ eşit olduğunu görürüz.

$(a-c)(b-a)\leq\left(\frac{(a-c)+(b-a)}2\right)^2=\frac{(b-c)^2}4\quad A.O\geq G.O$

$\frac{(a-c)+(b-a)}2\leq\left[\frac{(a-c)^2+(b-c)^2}2\right]^{\frac12}$ Kuvvet ??

$(b-c)^2\leq2[(a-c)^2+(b-a)^2+(b-c)^2]$ bulunur. Her iki tarafa $2(b-c)^2$ eklersek

$3(b-c)^2\leq 2[(a-c)^2+(b-a)^2+(b-c)^2]$

$\vert (a-c)(a-b)(b-c)(a+b+c)\vert\leq\frac14(b-c)^3(a+b+c)$ our. $(b-c)^2$ bildiğimiz için eşitsizliğin sağ tarafının önce karesini alıp sonra karekök alalım

$\vert (a-c)(a-b)(b-c)(a+b+c)\vert  \leq  \frac14\sqrt{(b-c)^6(a+b+c)^2} $

 $= \frac14\sqrt{\left( \frac{2[(b-a)^2+(a-c)^2+(c-b)^2]}3\right)^3(a+b+c)^2}$

$= \frac{\sqrt2}2\left(\sqrt[4]{\left(\frac{(b-a)^2+(a-c)^2+(c-b)^2}3\right)^3(a+b+c)^2}\right)^2$

4 terim olduğu için önce kare kökünü sonra sonra karesini aldık tekrar

$\leq \frac{\sqrt2}2\left[ \frac14(b-a)^2+\frac14(a-c)^2+\frac14(c-b)^2+\frac14(a+b+c)^2 \right]^2$ Ağırlıklı AO$\geq$ GO

$\leq \frac{\sqrt2}{32}\left[(b-a)^2+(a-c)^2+(c-b)^2+(a+b+c)^2\right]^2$

açarsak

$\leq \frac{\sqrt2}{32}\left(3(a+b+c)^2\right)^2$

$=\frac{9\sqrt2}{32}\left(a^2+b^2+c^2\right)^2$

$M=\frac{9\sqrt2}{32}$ bulunur.


(6.1k puan) tarafından 
tarafından düzenlendi
0 beğenilme 0 beğenilmeme

bir yerlerde hata yapmış olabilirim ama 9 kök2/32 buldum ama buraya yazmam çok kolay değil ancak fotoğrafını upload edebilirim

(1.8k puan) tarafından 
20,200 soru
21,728 cevap
73,275 yorum
1,887,854 kullanıcı